2
$\begingroup$

It was not hard to google that the simpler sum $\displaystyle \sum_{p < N} \frac{1}{p} = c + \log \log N $ so it is divergent.

The sum of reciprocals of primes squared also converges $\displaystyle \sum_p \frac{1}{p^2} = 0.4522\dots$


Does $\displaystyle \sum_{p} \frac{1}{p} \left\{ \frac{N}{p} \right\} $ stay bounded as $N$ gets large? What about $\displaystyle \sum_{p,q } \frac{1}{pq} \left\{ \frac{N}{pq} \right\} $ for large $N$?

Maybe it's necessary to say $p,q < N$. This is related to my earlier question on the Euler $\phi$-function

$\endgroup$
1
  • 3
    $\begingroup$ The first sum is asymptotic to $(1-\gamma)\log \log N$, where $\gamma$ is Euler's constant, and the second sum is asymptotic to $c_2(\log \log N)^2$ for some constant $c_2$. I am very curious though, how does this relate to your earlier question? $\endgroup$ May 3, 2012 at 18:21

0

Your Answer

By clicking “Post Your Answer”, you agree to our terms of service and acknowledge you have read our privacy policy.

Browse other questions tagged or ask your own question.